why isn’t -3 a whole number? please answer asap

Answers

Answer 1

Answer:

-3 is not a whole number because the whole number is like a zero , no fraction or demical part and no negative

Hope this helps :)

Step-by-step explanation:

-3 has negative sign so it's not a whole number

Answer 2

Answer:

-3 is not a whole number.

Whole number:

Starts from zero.Doesn't include negative integers.{0,1,2,3,4,5,6.....}

-3 is an integer, a negative integer.

Integer:

Subset of:Negative integersZeroNatural number, Whole number(positive integers as a whole)

Hope this helps ;) ❤❤❤


Related Questions

How do you find a square root of a number? PLease use 81 as an example.

Answers

You have to find the two number that you can multiply to get 81 but the answer is 9

Shelly is buying shoes online and computes that she has enough money to buy 5.91 pairs of shoes.
How many pairs of shoes can she buy?

Answers

Answer:

5

Step-by-step explanation:

This is because she cannot buy 5.91 pairs of shoes and she doesn't have enough money to buy exactly 6, so you must round it down to 5.

She can buy a maximum of 5 shoes with her money.

What is the reasoning?

The procedure of utilizing logical reasoning to analyze a condition to determine the best problem-solving approach for a particular issue, then using this approach to create and explain a resolution.

In another word, reasoning is a tricky but interesting problem in mathematics that required relations of variables to solve.

Given that,

Shelly has money to buy 5.91 pairs of shoes.

Now,

Shoes are a whole quantity it will never come in fractions or decimals.

So,

We need to write 5.91 either 5 or 6

Now,

If we write it 6 then we do have not sufficient money but if we write 5 then we have money although some amount will be left still, she can buy 5 shoes.

Hence "She can buy a maximum of 5 shoes with her money".

For more about reasoning,

https://brainly.com/question/2923977

#SPJ6

What is the degree for this

Answers

Answer: linear/first degree

Step-by-step explanation:

 (6-t-t^4)+(9t+t^4)

=6-t-t^4+9t+t^4

=8t+6

The degree of a polynomial is determined by the highest degree in the polynomial. The highest degree in this binomial is one, which is first degree or known as linear.

Hope this helps!! :)

Please let me know if you have any question

Express area of rectangle. Answer with a polonomial in stadard form. Image of rectangle below. Please answer, i will give brainiliest.

Answers

Answer:

Area = (x² + 11x + 24)

Step-by-step explanation:

Area of the blue square with sides 'x' units = (Side)²

                                                                       = x²

Area of the pink rectangle = Length × width

                                            = 3 × (x)

                                            = 3x

Area of the orange rectangle = Length × Width

                                                 = 8 × 3

                                                 = 24

Area of the green rectangle = Length × width

                                               = (x) × 8

                                               = 8x

Now total area of the entire rectangle = x² + 3x + 24 + 8x

                                                                = (x² + 11x + 24)

Therefore, area of the entire rectangle will be a polynomial (x² + 11x + 24)

Can someone help me with this problem

Answers

Increasing
Increasing
Decreasing
t=3
t=14
V=4
Answers:A) The graph of the function is increasing over the interval 0 < t < 3B) The graph of the function is increasing over the interval 14 < t < 18C) The graph of the function is decreasing over the interval 3 < t < 14D) The graph has a max value of V = 7 at t = 3E) The local min (V = 1) occurs when t = 14F) I'm not sure what your teacher is trying to say here. There isn't a local max or local min when t = 18. Seems like a typo. If you ignore the "local max" part and you're only wanting the value of V, then V = 4 here.

========================================================

Explanation:

The graph goes uphill as you read from left to right over the interval 0 < t < 3. This means t is between 0 and 3. So the graph is increasing over this interval. The same applies for the interval 14 < t < 18 as well.

The graph goes downhill we we are between t = 3 and t = 14, so that's why the graph is decreasing on the interval 3 < t < 14.

The local min is the lowest point of the graph. For sine functions like this, they occur infinitely many often. But we're restricting to just one cycle (so to speak) and we only have one local min at (t, V) = (14, 1). So that explains the answer to part E. The answer to part D is similar but we're looking at the highest point this time at (3,7)

As for part F, it seems like your teacher made a typo because the graph is neither at the highest nor the lowest point when t = 18. The graph is right in the middle instead.

Given directed line segment QS , find the coordinates of R
such that the ratio of QR to RS is 3:5. Plot point R

Answers

Answer:

The answer is below

Step-by-step explanation:

The question is not complete, what are the coordinates of point Q and R. But I would show how to solve this.

The location of a point O(x, y) which divides line segment AB in the ratio a:b with point A at ([tex]x_1,y_1[/tex]) and B([tex]x_2,y_2[/tex]) is given by the formula:

[tex]x=\frac{a}{a+b}(x_2-x_1)+x_1\\ \\y=\frac{a}{a+b}(y_2-y_1)+y_1[/tex]

If point Q is at ([tex]x_1,y_1[/tex]) and S at ([tex]x_2,y_2[/tex])  and R(x, y) divides QS in the ratio QR to RS is 3:5, The coordinates of R is:

[tex]x=\frac{3}{3+5}(x_2-x_1)+x_1=\frac{3}{8}(x_2-x_1)+x_1\\ \\y=\frac{3}{3+5}(y_2-y_1)+y_1=\frac{3}{8}(y_2-y_1)+y_1[/tex]

Let us assume Q(−9,4) and S(7,−4)

[tex]x=\frac{3}{8}(7-(-9))+(-9)=\frac{3}{8}(16)-9=-3\\\\y=\frac{3}{8}(-4-4)+4=\frac{3}{8}(-8)+4=1[/tex]

Answer: It’s 2.6

Step-by-step explanation:

11\14 plus 2\7 and since it makes me write extra characters here u go

Answers

15/14 or 1 1/14
Just change 2/7 to 4/14 and add.
11 2 (2)
—- + —- —-
14 7 (2)

11 + 4
—- —-
14 14

15 1
—- 1—
14 or 14

What equation do you get when you solve a-q=a+sx for x?

Answers

Answer:

[tex]x=\frac{-q}{s}[/tex]

Step-by-step explanation:

[tex]a-q=a+sx\\a-a-q=a-a+sx\\-q=sx\\\frac{-q}{s}=\frac{sx}{s}\\x=\frac{-q}{s}[/tex]

A triangle has side lengths of (1.4u+9.6)(1.4u+9.6) centimeters, (4.3u-9.9)(4.3u−9.9) centimeters, and (5.3v+1.8)(5.3v+1.8) centimeters. Which expression represents the perimeter, in centimeters, of the triangle?

Answers

Answer:

5.7u+5.3v+1.5

Step-by-step explanation:

Given a triangle with side lengths s1, s2 and s3, the perimeter of the triangle in centimetre is expressed as the sum of all the sides of the triangle i.e

Perimeter of a triamgle = s1+s2+s3

Given the sides of the triangle in question as (1.4u+9.6) centimeters, (4.3u-9.9)centimeters, and (5.3v+1.8) centimeters, the perimeter of the triangle will be the sum of all the functions as shown;

Let s1 = (1.4u+9.6)cm

s2 = (4.3u-9.9)cm

s3 = (5.3v+1.8) cm

Perimeter of the triangle = 1.4u+9.6 + 4.3u-9.9 + 5.3v+1.8

collect like terms

P =  1.4u+ 4.3u+5.3v+9.6-9.9 +1.8

P = 5.7u+5.3v+1.5

Hence the expression that represents the perimeter of the triangle is 5.7u+5.3v+1.5


There are 12 inches in 1 foot and 5,280 feet in 1 mile. Elena ran miles.



1. How many feet is that

Answers

Answer: 5,280

Step-by-step explanation:

hello please help!! thank you<3

Answers

Answer:

112

Step-by-step explanation:

Match the name with the correct figure help please!!! ​

Answers

Answer:

B Is the right one that the answer

B is the correct answer. Just finished that quiz

A store purchased a washing machine and marked it up 20% from the original cost of
$300. Then, wanting to make room for summer inventory, the store placed the washing
machine on sale for 75% off. What was the price after the discount?

Answers

Answer:

The answer is $270

Step-by-step explanation:

7.
Which point on the number line best represents the location of V92?
MN
Q
+
8
9
10
A. Point M
B. Point N
C. Point P
D. Point Q

Answers

Answer:

C

Step-by-step explanation:

Square root of 92 is 9.591663047

so round that to 9.59

3/2(6x+4)=12 im dumb help me

Answers

Answer:

The correct answer is x = 2/3.

Step-by-step explanation:

To solve this problem, the first step I would take is multiply both sides by 2.  This will allow us to get rid of the fraction on the left side of the equation and make it easier to solve.  This is modeled below:

3/2(6x + 4) = 12

3(6x +4) = 24

Next, we can use the distributive property on the left side of the equation to get rid of the parentheses.

18x + 12 = 24

Then, we can subtract 12 from both sides of the equation.

18x = 12

Finally, we can divide both sides by 18 and simplify.

x = 12/18 = 2/3

Therefore, the correct answer is x =2/3.

Hope this helps!

A line segment starts at the point (2,3) and ends at the point (6,11). Find the equation of the perpendicular bisector of the segment. Demonstrate that your answer is correct. Your equation must be written in slope intercept form.​

Answers

1). Slope = (-2)

2). Midpoint = (2, -2)

3). Slope of the perpendicular bisector = (1/2)

4). Equation of perpendicular bisector will be x - 2y = 6

Step-by-step explanation:

A line segment has the endpoints at (4, -6) and (0, 2).

1). Then the slope of the given line segment will be = (y - y')/(x - x') = (2 + 6)/(0 - 4) = 8/(-4) = (-2)

2). Mid point of the line segment is given by

Therefore midpoints of the line segment will be  = (2, -2)

3). Slope of the perpendicular bisector is represented by

⇒ (-2)×m2 = (-1)

4). Now we have to find the equation of perpendicular bisector passing through (2, -2) and slope (1/2).

Since standard equation of the line will be given as y = mx + c

passes through (2, -2).

c = (-1) - 2 = -3

Finally the equation of perpendicular bisector will be

⇒ 2y = x - 6

⇒ 2y - x = -6

⇒ x - 2y = 6

Can you write a rule for finding the output (absolute guessing error) given the input (a guess)?

Answers

Answer:

The rule for the plotted line is [tex]y = \left | x - 50 \right |[/tex]

Step-by-step explanation:

The coordinates when y = 0, x = 50

When y = 10, x = 60

Therefore, the slope = (10 - 0)/(60 - 50) = 1

The equation in slope and intercept form is y - 10 = 1 × (x - 60)

y = x -60 + 10 = x -50

y = x - 50

Also, we have

When y = 20, x = 30

y = 10, x = 40

Therefore, the slope = (10 - 20)/(40 - 30) = -1

The equation in slope and intercept form is y - 10 = -1 × (x - 40)

y = -x + 40 + 10

y = 50 - x

Therefore, when x < 50, y = 50 - x

When x > 50, y = x - 50

Which is equivalent to [tex]y = \left | x - 50 \right |[/tex]

The rule for the plotted line is [tex]y = \left | x - 50 \right |[/tex]

A shop buys a pair of shoes for $50. They sell them for $60. What is the mark–up on the shoes? in dollars

Answers

Answer:

the mark-up is $10 or 20% of the original price.

Step-by-step explanation:

This is because 60-50 is 10. And 20% or 1/5 of 50 is 10.

a point where a graph crosses the y axis is called a​

Answers

Answer:

y intercept

Step-by-step explanation:

The point in which the graph crosses the x-axis is called the x-intercept and the point in which the graph crosses the y-axis is called the y-intercept.

The lowest recorded temperature in a town was 4ºF, without the windchill. With the wind, the temperature felt like 10ºF below zero. On a number line, what is the distance between the two temperatures?

Answers

Given that the temperature of a town was 4ºF without the wind, and later 10ºF below zero with the wind, the distance between the two temperatures of the town on a number line is: 14 units.

The record of temperature of the town can be illustrated or plotted on a number line as shown in the diagram attached below.

The temperature of the town at 4ºF will be located 4 units away from point zero on the number line.

The temperature of the town at 10ºF below zero will be located 10 units away from point zero on the number line.

Thus the distance between these two temperatures will be:

4 units + 10 units = 14 units.

Therefore, given that the temperature of a town was 4ºF without the wind, and later 10ºF below zero with the wind, the distance between the two temperatures of the town on a number line is: 14 units.

Learn more here:

https://brainly.com/question/17843289

What type of number is 33/3?

Answers

Answer:

A Fraction

Step-by-step explanation:

33/3 is a fraction because it consists of a numerator and a denominator.

33/3 is a fraction.

i need help asap please help

Answers

Answer:

2.33333333333333333 (two point repeating three)

The cost of admission to a popular music concert was $122 for 11 children and 3 adults. The admission was $102 for 6 children and 4 adults in for another group. How much was the admission for each child and adult?

Answers

Answer:

Step-by-step explanation:

whats the value of the expression3-⁴

Answers

Answer:

1/81

Step-by-step explanation:

3^-4 = -81

Put this in a fraction: 1/81

Hope this helps you!

0.7 rational or irrational

Answers

0.7 is Rational not irrational

Answer:

it is a rational number.

Step-by-step explanation:

It is read as seven tenths and is equivalent to the fraction 7/10. Since it can be written as a fraction, it is rational.

Simplify the Radical Expression
3V2 - V8 +4V12+53

Answers

The answer is the other one V2+13

8x + 3 (x +1) = 5x - (9x – 6)

Answers

Answer:

x=1/5

Step-by-step explanation:

1/5 < that’s the answer

What is the irrational number to the nearest hundedth of the square root of 32.​

Answers

Answer:

5.66

Step-by-step explanation:

it would round up to 5.66

4. Which step will solve the equation below for x? *
1 point
x2 = 64
Taking the square root of both sides of the equation
Taking the cube root of both sides of the equation
Dividing both sides of the equation by 2
Subtracting 2 from both sides of the equation

Answers

Answer:

D

Step-by-step explanation:

You should know this its not hard

I wish I could help but I can’t calculate the right answer

Find the value of 2.
A.126
B.121.5
C.117
D.63

Answers

Answer:

D

Step-by-step explanation:

The tangent- tangent angle x is half the difference of the intercepted arcs.

The sum of the 2 arcs on the circle = 360°, thus

lower arc = 360° - 243° = 117°, then

x = [tex]\frac{1}{2}[/tex] (243 - 117)° = 0.5 × 126° = 63°

Other Questions
Which verb form correctly completes this sentence? Sus padres ________ en los Estados Unidos durante dos aos. A. vivieron B. vivimos C. viviste D. vivi Glucose molecules provide energy to power the swimming motion of sperm. In this example, the sperm are changing Need help asap! HDTV screens usually have a width-to-height ratio of 16:9. Which screen dimensions are in the same proportion? 26in by 19in, 48cm by 27cm, 4ft by 3ft,28in by 21in Coronary artery disease is caused by the _______ build up in the wall of the arteries possibly blocking the flow of blood and oxygen to the heart. Which of the following was the purpose of the chorus in ancient Greek theater? Marcie solved the following inequality, and her work is shown below:2(x 5) 6x + 18 2x + 10 6x + 18 8x +10 18 8x 8 x 1What mistake did Marcie make in solving the inequality? Which region of the country is most accommodating of voters, in that it is overall easier to vote early, by mail, and by absentee ballot? Help me please please please The measure of judicial action intended to compensate an injured party in a civil lawsuit is known as a(n) A sample initially contains 4.4 moles of a radioactive isotope. How much of the sample remains after four half-lives? "Every now and then, groundbreaking ideas or studies come along, such as Growth Mindset." What does the word groundbreaking mean in this sentence? A.) Innovative B.)Tired C.) Redundant D.) Specific Counterspy in a sentence. Not the definition! Prove the proposition p (0), where p (n) is the proposition "if n is a positive integer greater than 1, then n2 > n." what kind of proof did you use? what is the difference between deep water and shallow water waves??? What personal strengths have you discovered through your internship experience? "Why We Should Work Less" Adapted from Richard Schiffman Why is it a good idea for the HR plan to consider changes recruiters have seen in the companys workforce? one reason france sent explores to north america in early 1500's was to Donna Divider sees the Multiples of 5 list and says, Multiples of 5 are all divisible by 5! They can all be divided by 5 with no remainder! Is Donna correct? Explain why this is so. Flocked in the lesson summary means